Difference between revisions of "1999 AHSME Problems/Problem 20"

m (See also)
(Solution)
Line 12: Line 12:
 
Realizing this, one can easily prove by induction that <math>\forall n\geq 3;~ a_n=m</math>.
 
Realizing this, one can easily prove by induction that <math>\forall n\geq 3;~ a_n=m</math>.
  
It follows that <math>m=a_9=99</math>. From <math>19=a_1=m-x</math> we get that <math>x=80</math>. And thus <math>a_2 = m+x = \boxed{179}</math>.
+
It follows that <math>m=a_9=99</math>. From <math>19=a_1=m-x</math> we get that <math>x=80</math>. And thus <math>a_2 = m+x = \(E)boxed{179}</math>.
  
 
== See also ==
 
== See also ==

Revision as of 21:44, 14 December 2018

Problem

The sequence $a_{1},a_{2},a_{3},\ldots$ statisfies $a_{1} = 19,a_{9} = 99$, and, for all $n\geq 3$, $a_{n}$ is the arithmetic mean of the first $n - 1$ terms. Find $a_2$.

Solution

Let $m$ be the arithmetic mean of $a_1$ and $a_2$. We can then write $a_1=m-x$ and $a_2=m+x$ for some $x$.

By definition, $a_3=m$.

Next, $a_4$ is the mean of $m-x$, $m+x$ and $m$, which is again $m$.

Realizing this, one can easily prove by induction that $\forall n\geq 3;~ a_n=m$.

It follows that $m=a_9=99$. From $19=a_1=m-x$ we get that $x=80$. And thus $a_2 = m+x = \(E)boxed{179}$ (Error compiling LaTeX. Unknown error_msg).

See also

1999 AHSME (ProblemsAnswer KeyResources)
Preceded by
Problem 19
Followed by
Problem 21
1 2 3 4 5 6 7 8 9 10 11 12 13 14 15 16 17 18 19 20 21 22 23 24 25 26 27 28 29 30
All AHSME Problems and Solutions

The problems on this page are copyrighted by the Mathematical Association of America's American Mathematics Competitions. AMC logo.png